Sie sind auf Seite 1von 68

Richard F. Daley and Sally J.

Daley
www.ochem4free.com

Organic
Chemistry
Chapter 12

Aliphatic Nucleophilic Substitution


12.1 Naming Single Bonded Heteroatom Functional
Groups
579
12.2 Comparing Nucleophilic Substitution Reaction
Mechanisms 586
12.3 The SN1 and SN2 Reaction Mechanisms
12.4 Stereochemistry of Nucleophilic Substitutions
12.5 The Substrate
595
12.6 Nucleophiles and Leaving Groups
601
12.7 Common Nucleophiles
606
12.8 The Reaction Medium
607
12.9 SN1 versus SN2
613
12.10 Halide Nucleophiles
613
Synthesis of 1-Bromobutane616
12.11 Oxygen Nucleophiles
620
12.12 Nitrogen Nucleophiles
625
Synthesis of 2,5-Diaminoadipic Acid
628
12.13 Carbon Nucleophiles
631
12.14 Neighboring Group Participation
634
Special Topic - SN1 vs. SN2
637
Key Ideas from Chapter 12
641

588
592

Organic Chemistry - Ch 12

578

Daley & Daley

Copyright 1996-2005 by Richard F. Daley & Sally J. Daley


All Rights Reserved.
No part of this publication may be reproduced, stored in a retrieval system, or
transmitted in any form or by any means, electronic, mechanical, photocopying,
recording, or otherwise, without the prior written permission of the copyright
holder.

www.ochem4free.com

5 July 2005

Organic Chemistry - Ch 12

579

Daley & Daley

Chapter 12

Aliphatic Nucleophilic Substitution


Chapter outline
12.1

Naming Single Bonded Heteroatom Functional


Groups
Naming alcohols, ethers, phenols, thiols, thioethers, sulfides,
and amines

12.2

Comparing Nucleophilic Substitution


Reaction Mechanisms
A comparison of the mechanism for SN1 and SN2 reactions with
the mechanism of nucleophilic substitution at carbonyl groups

12.3

The SN1 and SN2 Mechanisms

More details for the SN1 and SN2 reaction mechanisms

12.4

Stereochemistry of Nucleophilic Substitutions


The stereochemistry of the SN1 and SN2 reaction mechanisms

12.5

The Substrate
Effect of substrate structure on the outcome of nucleophilic
substitution reactions

12.6

Nucleophiles and Leaving Groups

12.7

Common Nucleophiles

The factors that make good nucleophiles or leaving groups


A listing of some of the common nucleophiles used in organic
chemistry

12.8

The Reaction Medium


Effect of the solvent on the outcome of nucleophilic substitution
reactions

12.9

SN1 versus SN2


Factors that influence whether an E1 or an E2 mechanism will
prevail

12.10

Halide Nucleophiles
Substitution reactions with halide nucleophiles

12.11

Oxygen Nucleophiles

12.12

Nitrogen Nucleophiles

12.13

Carbon Nucleophiles

Reactions using oxygen nucleophiles


Using nitrogen nucleophiles in substitution reactions
Substitution reactions with carbon nucleophiles derived from
Grignard reagents and alkyllithium reagents

12.14

Neighboring Group Participation


How a group adjacent to a functional group affects the outcome
of reactions

www.ochem4free.com

5 July 2005

Organic Chemistry - Ch 12

580

Daley & Daley

Objectives
Apply the IUPAC rules of nomenclature to naming alcohols, thiols,
ethers, sulfides, and amines
Be able to write the mechanisms for both the SN1 and SN2 reaction
types
Know the effect of the SN1 and SN2 reaction mechanisms on the
stereochemistry of reactions
Recognize how the structure of the substrate affects the outcome of
nucleophilic reactions
Understand the factors that affect the stability of carbocations
Recognize the factors that make good nucleophiles and good
leaving groups
Know how the solvent affects the outcome of nucleophilic
substitution reactions
Understand how halide, oxygen, nitrogen, and carbon nucleophiles
react with various substrates
Know how a functional group adjacent to the site of a nucleophilic
substitution affects the course of reactions

If there was two birds sitting on a fence, he would bet


you which one would fly first.
- Mark Twain

liphatic nucleophilic substitution is the third


installment in the study of nucleophilic reactions in
organic chemistry. The previous two installments, nucleophilic
addition and nucleophilic substitution at the carbonyl group, describe
the formation of new bonds to the carbonyl carbon of aldehydes,
ketones, and the carboxylic acid family. Chapters 7 and 8 discuss the
nature of the substituents bonded to the carbonyl carbon and how they
determine whether a substitution or an addition reaction takes place.
This chapter presents the study of aliphatic nucleophilic
substitution reactions at sp3 hybridized carbon atoms. The outcome of

www.ochem4free.com

5 July 2005

Organic Chemistry - Ch 12

581

Daley & Daley

aliphatic nucleophilic substitution reactions depends on such variables


as nucleophilicity, stereochemistry, the reaction medium, and the
intermediate's stability. Aliphatic nucleophilic substitution reactions
have two, often competing, reaction mechanisms. These two
mechanisms account for a large variety of reactions in organic
chemistry. Although the two mechanisms are different, they both have
a nucleophile, a leaving group, and a substrate; and all three play an
important part in the outcome of each reaction. An understanding of
the characteristics of these two mechanisms gives you a broader
understanding of organic chemistry. What you learn about these two
mechanisms will help you later with further organic syntheses and
mechanisms.
Through their work with aliphatic nucleophilic substitutions,
chemists have perhaps learned more about organic reaction
mechanisms than through their study of any other reaction type.
However, these gains did not come easily. From the early years of the
twentieth century until the 1970s, the debate raged over the issues
surrounding aliphatic nucleophilic substitution. This debate included
such things as experimental results that various investigators
interpreted in opposite ways and experimental results that all agreed
seemed mutually incompatible. Fortunately, most of these questions
are now resolved.

12.1 Naming Single Bonded Heteroatom


Functional Groups
This section covers the nomenclature of compounds that
contain a singly bonded oxygen, sulfur, or nitrogen; that is, alcohols,
ethers, phenols, thiols, thioethers (or sulfides), disulfides, and amines.
Each of these functional groups replaces a hydrogen on the carbon
skeleton of the organic compound. Each functional group also changes
the suffix of the parent name in a way characteristic of that group.
The parent compound, in most cases, includes the carbon in the main
chain that bears the functional group.
Alcohols replace a hydrogen on the parent carbon chain with an
OH group. The suffix for an alcohol is ol in the IUPAC
nomenclature. When naming an alcohol, follow these steps.
Step 1 Determine the parent chain.
Step 2 Drop the final e of the parent hydrocarbon name and add
the ol ending.
Step 3 To number substituents, number the carbon chain to give
the carbon bearing the OH group the lowest possible number.

www.ochem4free.com

5 July 2005

Organic Chemistry - Ch 12

582

Daley & Daley

OH
CH3OH
Methanol

CH3CH2OH
Ethanol

CH3CH2CHCH3
2-Butanol

OH

OH

CH3
2-Methylcyclohexanol

A polyol is a molecule
that contains two or
more OH groups.

Primary, secondary,
and tertiary alcohols
have one, two, or three
alkyl groups attached
to the carbon bearing
the OH group.

Frequently, chemists call the OH group a hydroxy group and


compounds that contain two or more hydroxy groups polyols. To
name polyols, use the IUPAC suffixes of diol, triol, and so forth. 1,2Ethanediol (HOCH2CH2OH) is an example of the proper naming of a
polyol.
Alcohols can be classified according to the number of carbon
atoms bonded to the carbon that bears the alcohol group. For example,
ethanol is a primary alcohol because the carbon bearing the hydroxy
group is attached to only one other carbon. 2-Propanol is a secondary
alcohol because the carbon bearing the OH group is bonded to two
other carbons, and 2-methyl-2-propanol is a tertiary alcohol because
the OH bearing carbon is bonded to three other carbons.

OH
A primary alcohol

OH

OH

Ethanol

Benzyl alcohol is a
carbon attached to a
benzene ring and an
OH group.

3-Methyl-2-butanol

2-Propanol

2-Methyl-2-propanol

A secondary alcohol

A tertiary alcohol

Benzyl alcohol is alcohol attached to a carbon that also bears


a phenyl group.
OH

OH
I

Benzyl alcohol

4-Iodobenzyl alcohol

www.ochem4free.com

5 July 2005

Organic Chemistry - Ch 12

A phenol is a benzene
ring substituted with
an OH group.

583

Chemists call a hydroxy group connected to a carbon atom of a


benzene ring a phenol after the simplest member of this group,
hydroxybenzene or, more commonly, phenol. To name substituted
phenols, number the ring so that the carbon bearing the OH group
is C1. Because the common name for an alcohol is alkyl alcohol (i.e.,
ethanol is often called ethyl alcohol) many organic chemistry students
are tempted to confuse benzyl alcohols with phenols. Be careful!
Phenol nomenclature is complicated by the fact that many
phenols have common names that are more widely used than the
IUPAC names. For example, 2-methylphenol is known as o-cresol, 3methylphenol is known as m-cresol and 4-methylphenol is known as pcresol.
OH

Phenol

A thiol, also called a


mercaptan, contains an
SH group.

OH

Cl

CH3
3-Methylphenol
(m-Cresol)

OH

3-Chlorophenol
(m-Chlorophenol)

Chemists call the sulfur analogs of the alcohols thiols, or


mercaptans. To name thiols, add the suffix thiol to the parent
name. However, unlike most suffixes, the thiol suffix does not replace
the final e of the hydrocarbon name.
CH3SH
Methanethiol

A thiophenol is the
sulfur analog of a
phenol.

Daley & Daley

CH3CH2SH
Ethanethiol

Thiols have a characteristic intensely unpleasant odor. Some common


examples are the defensive odor of the skunk and the characteristic
odors of garlic and onion. Replacing the oxygen in a phenol with a
sulfur creates a class of compounds called thiophenols. Name
thiophenols following the same rules for naming phenols.
Exercise 12.1
Name the following compounds.

www.ochem4free.com

5 July 2005

Organic Chemistry - Ch 12

584

a)

Daley & Daley

b)

OH

OH

c)

d)
Cl

OH

SH

e)

f)
OH

SH

Sample solution
b) The common name for this compound is menthol. Menthol is
extracted from the oil of peppermint. Its IUPAC name is 2-methyl-5(1-methylethyl)cyclohexanol.
An ether contains two
alkyl or aryl groups
bonded to an oxygen.

An alkoxy group is
derived from an
alcohol with some
substituent replacing
the proton on the
oxygen.

Ethers are compounds with two alkyl or aryl groups bonded to


the same oxygen atom. Unfortunately, nomenclature for ethers is
somewhat confusing because the IUPAC has not adopted a specific
system for naming them. Currently, there are two systems that
chemists usually use. One system names the least complicated of the
alkyl groups bonded to the oxygen, plus the oxygen atom (RO), as an
alkoxy group, and the other alkyl group as the parent compound. The
other system names the compound as an alkyl alkyl ether or as a
dialkyl ether. This system calls both alkyl groups bonded to the
oxygen alkyl substituents of the ether. Here are some examples.
CH3
CH3OCH

CH3CH2OCH2CH3

CH3

www.ochem4free.com

5 July 2005

Organic Chemistry - Ch 12

585

Daley & Daley

2-Methoxypropane
or
isopropyl methyl ether

Ethoxyethane
Or
Diethyl ether

In general, when the two alkyl groups are of similar size, use the alkyl
alkyl ether system; when one group is larger than the other, use the
alkoxyalkane name.
For cyclic ethers, the most common systematic name counts the
oxygen as if it were a carbon in the ring and adds an oxa prefix to the
parent name. This prefix is rarely used with acyclic ethers. When
numbering cyclic ethers, begin with the ether oxygen. An example of a
cyclic compound is a three-membered ring with one oxygen and two
carbons. It is named oxacyclopropane. The other examples shown
below have their common names in parentheses.
O

O
Oxacyclopentane
(Tetrahydrofuran)
Sulfides are the sulfur
analogs of an ether.
Alkylthio groups are
sulfur analogs of
alkoxy groups.
Disulfides contain two
sulfurs bonded
together.

1,4-Dioxacyclohexane
(1,4-Dioxane)

Sulfides, or alkylthio compounds, are sulfur analogs of


ethers. The alkylthio name is comparable to the alkoxy name. A
number of sulfides have two sulfur atoms bonded together with the
formula RSSR. Chemists call these compounds disulfides.
CH3SCH(CH3)2

CH3CH2SCH2CH3

2-Methylthiopropane
or
Isopropyl methyl sulfide

Ethylthioethane
or
Diethyl sulfide

S
S

Dicyclohexyl disulfide

Exercise 12.2
Using IUPAC nomenclature, name the following compounds.

www.ochem4free.com

5 July 2005

Organic Chemistry - Ch 12

586

Daley & Daley

b)

a)
(CH3)3COCH3

d)

c)
O

SSCH3

(As a substituent, an
ethenyl group is often
called a vinyl group)
e)

f)
S

Sample solution
e) The name of this cyclic ether is 3-(1-methylethyl)oxacyclopentane.
Most chemists call this 3-isopropyltetrahydrofuran.
Amines consist of a
nitrogen bonded to a
combination of three
carbon or hydrogen
substituents.

Amines are organic derivatives of ammonia (NH3) with one,


two, or three alkyl groups to replace the hydrogens bonded to the
nitrogen. Chemists classify these derivatives as primary, secondary, or
tertiary amines, respectively. The terms primary, secondary, and
tertiary have somewhat different meanings for amines than they do
for alcohols. Ammonia with one hydrogen replaced by an alkyl group
(RNH2) is a primary amine; with two hydrogens replaced, it is a
secondary amine (R2NH); and with all three replaced, it is a tertiary
amine (R3N).
CH3NH2
Methanamine
or
Methylamine
(A primary amine)

(CH3)2NH
N-Methylmethanamine
or
Dimethylamine
(A secondary amine)

(CH3)3N
N,N-Dimethylmethanamine
or
Trimethylamine
(A tertiary amine)

To name amines using IUPAC nomenclature, follow these


steps.

www.ochem4free.com

5 July 2005

Organic Chemistry - Ch 12

587

Daley & Daley

Step 1 Determine the parent compound.


Step 2 Drop the final e of the alkane name of the parent
hydrocarbon and add the suffix amine.
A second system for naming amines uses the suffix ylamine
for naming amines with similar hydrocarbon substituents. For
example, CH3NH2 is called methylamine.
Step 3 Name the alkyl groups of secondary and tertiary amines as
substituents to the parent compound name by adding the letter
N to show that the group is bonded to the nitrogen atom.
a) For secondary amines, add one N.
b) For tertiary amines, add two Ns.
Step 4 If the molecule contains a second nitrogen, denote any
substituents on that nitrogen with the letter N'.
Here are some examples of amines.
N(CH3)2
(CH3)2NCH2CH(CH3)2
N,N,2-Trimethylpropanamine
N,N-Dimethylcyclohexanamine
CH3NHCH2CH2N(CH3) 2
N,N,N'-Trimethylethanediamine

Step 5 Use the prefix amino, or alkylamino, when the amine is a


substituent on a parent molecule with another functional
group. For example, when an amine and an alcohol are in the
same molecule, the alcohol group determines the parent name
and either of the prefixes amino or alkylamino names the
amine as a substituent group.
N(CH3)2

OH
2-(N,N-Dimethylamino)cyclopentanol

Step 6 For some amines, particularly the cyclic amines, chemists


sometimes use the prefix aza, which is similar to the oxa
prefix of ethers. The name for an amine group attached to a
carbon atom of a benzene ring is aniline.

www.ochem4free.com

5 July 2005

Organic Chemistry - Ch 12

588

Daley & Daley

NH2

N
H
Azacyclohexane
(Piperidine)

Aniline

Exercise 12.3
Using IUPAC nomenclature, name the following compounds.
a)

b)
CH2CH3

(CH3)3CCH2NH2

CH3NCH2CH2CH3

c)

d)
NH 2
N(CH2CH3)2

e)

f)
NH2CH2CH2CH2CH2CH2NH2

H2C

CHCH2CH2NH2

Sample solution
b) N-Ethyl-N-methyl-1-propanamine

12.2 Comparing Nucleophilic Substitution Reaction


Mechanisms
Recall from Chapter 8 that in a nucleophilic substitution
reaction the nucleophile reacts with the substrate to displace the
leaving group. The carbonyl carbon of a carboxylic acid, or one of its
derivatives, is the obvious site for the reaction of a nucleophile
because the carbonyl carbon is both positively polarized and
unsaturated. Being positively polarized and unsaturated allows the
carbonyl carbon to readily accept the bonding electron pair from the
nucleophile. The sp3 hybridized carbon atoms also undergo
nucleophilic substitution but for different reasons.
Nucleophilic substitution reactions with sp3 hybridized carbon
atoms take place because of the electronegativity difference between

www.ochem4free.com

5 July 2005

Organic Chemistry - Ch 12

589

Daley & Daley

the leaving group and the carbon that it is bonded to. The leaving
group is a heteroatom with a greater electronegativity than carbon.
During a nucleophilic substitution reaction, the heteroatom departs
with a pair of electrons formerly bonded to the carbon atom. Because
the bond between an sp3 hybridized carbon and the leaving group is
polar and because the leaving group is more electronegative than
carbon, the carbon possesses a partial positive character. Thus, the
carbon is an electrophile and is reactive towards a nucleophile.
Although the fundamental reactions of a nucleophilic
substitution of a carbonyl carbon and an sp3 hybridized carbon are the
same, the mechanisms are not. With a carbonyl group, the S bond
between the carbon and oxygen shifts making room for an incoming
nucleophile to bond to the carbonyl carbon before the leaving group
departs. An analogous nucleophilic substitution at an sp3 carbon
would proceed as follows:

Nu

An aliphatic
nucleophilic
substitution reaction
occurs when an sp3
hybridized substrate
reacts with a
nucleophile.
A carbocation is a
positively charged
substrate with the
charge on a carbon
atom.
The term SN1 means
Substitution,
Nucleophilic,
Unimolecular

Nu

Nu

+ L

However, this mechanism leads to an intermediate with five bonds to


the carbon, which violates the octet rule. Thus, with an sp3 hybridized
carbon, the nucleophile cannot first add to the hybridized carbon and
then have the leaving group depart. It must follow some other
mechanistic pathway.
Aliphatic nucleophilic substitution reactions follow one of
two mechanisms. Which mechanism a particular reaction takes
depends, in part, on the nature of the substrate. The difference
between the two mechanisms is when the leaving group departs. With
the first mechanism, the leaving group departs before the nucleophile
arrives. In the second mechanism, the leaving group departs while the
nucleophile is reacting.
The first of these two reaction mechanisms begins with a
heterolytic cleavage of the bond between the sp3 carbon and the
leaving group in a heterolytic bond cleavage operation. The sp3 carbon
is then electron-deficient. With the leaving group and a pair of
electrons gone, the substrate is a positively charged carbocation. The
nucleophile then reacts with this carbocation in a heterogenic bond
forming the product. Chemists call a reaction that follows this
mechanistic pathway an SN1 reaction.

www.ochem4free.com

5 July 2005

Organic Chemistry - Ch 12

590

Nu +

+ L

Nu

Daley & Daley

Slow step

Fast step

The SN1 reaction mechanism

The term SN2 means


Substitution,
Nucleophilic,
Bimolecular.
A concerted reaction
involves simultaneous
bond making and bond
breaking.

In a reaction that follows the second reaction mechanism


pathway, the leaving group departs as the nucleophile arrives.
Simultaneously, the bond between the carbon and the leaving group
breaks, and the bond between the carbon and the nucleophile forms.
Chemists call this type of reaction mechanism an SN2 reaction. An
SN2 reaction mechanism describes a concerted reaction with a 1,3electron pair displacement operation.

Nu

Nu

Nu

+ L

The SN2 Reaction Mechanism

12.3 The SN1 and SN2 Reaction Mechanisms


2-Chloro-2-methylpropane (tert-butyl chloride) reacts with
water using acetone as the solvent to produce 2-methyl-2-propanol
(tert-butyl alcohol). In this reaction, a hydroxy group substitutes itself
for the chlorine of the substrate. The reaction follows the SN1
mechanistic pathway.
H3C
H3C

Cl

CH3
tert-Butyl chloride

H2O
acetone

H3C
H3C

OH

CH3
tert-Butyl alcohol

Chemists used this reaction to learn about the mechanism of


the SN1 reaction. They chose this particular reaction because of the
relative ease of using it to perform kinetic studies. A typical
experiment involved the reaction of the chloride with water at a
carefully controlled temperature. Chemists then followed the reaction
progress by determining the amount of HCl produced.

www.ochem4free.com

5 July 2005

Organic Chemistry - Ch 12

The rate for a first


order reaction depends
only on the
concentration of one
reagent in the reaction
mixture.
In a unimolecular
reaction the rate
depends on the
concentration of only
one chemical species.

591

Daley & Daley

As they studied the results of this experiment, they learned


that the rate of the reaction depended only on the concentration of the
tert-butyl chloride. Thus, the rate-determining step of the reaction
involved only the substrate and not the nucleophile. Chemists call
such a reaction a first order reaction. Below is the rate equation for
a first order reaction.
rate = k1[(CH3)3CCl]
This rate equation gives the mechanism its designation of SN1, or a
Substitution, Nucleophilic, unimolecular (1) reaction.
The separation of charge that occurs in the heterolytic bond
cleavage step makes it the rate-determining step in an SN1 pathway.
Thus, the two-step process fits better with first order kinetics.
Applying the SN1 mechanism from Section 12.2 to the reaction of tertbutyl chloride with water gives the following mechanism:
H3C
H3C

CH3

Cl

CH3

H2O

H3C

H3C CH3

H3C

Figure 6.4 is on page


000.

C
CH3

Cl

H3C
+

H3C

Slow

H3C CH3

CH3
C

O
H

O
H

CH3

Fast

H3C
H3C

OH

Fast

CH3

The reaction profile, which looks similar to the one in Figure


6.4, shows this three-step process. Notice that the rate-determining
step is the ionization of the carbonchlorine bond and that the rate
depends only on the concentration of the substrate. The carbocation
appears at the energy minimum between the transition state coming
from the substrate and the transition state leading to the product,
making it an intermediate product.

www.ochem4free.com

5 July 2005

Organic Chemistry - Ch 12

592

H3C
C
H3C
H3C

Daley & Daley

Cl

H3C
OH2

C
H3C
H3C

H3C
H3C C
H3C

CH3
C
H3C
H3C
H3C

O
H

Cl

H3C CH3
C

Cl + H2 O

H3C
OH2 + Cl

H3C C
H3C

H3C
H3C C
H3C

OH + HCl

Reaction Progress
Figure 12.1. Reaction profile for the SN1 reaction.

Exercise 12.4
Explain these observations about the reaction of tert-butyl chloride
with water to form tert-butyl alcohol.
a) Adding hydroxide ion, a better nucleophile than water, to the
reaction mixture makes little difference in the rate of reaction.
b) However, adding chloride ion to the reaction mixture markedly
slows down the rate of reaction.
The reaction of 1-chloropropane with water to form 1-propanol
follows the SN2 reaction mechanism. When using acetone as the
solvent, the rate of reaction is slow. However, adding hydroxide ion to
the mixture increases the rate of reaction. These data are typical of an
SN2 reaction mechanism.
CH3CH2CH2Cl

OH

CH3CH2CH2OH
1-Propanol

www.ochem4free.com

5 July 2005

Organic Chemistry - Ch 12

The rate for a second


order reaction depends
on the concentrations
of two reagents in the
reaction mixture.

In a bimolecular
reaction the rate
depends on the
concentrations of two
chemical species.

593

Daley & Daley

The rate of this reaction depends on the concentration of both the


alkyl chloride and the hydroxide ion. The kinetics make it a second
order reaction, because changes in the concentrations of both
reactants affect the rate of reaction.
rate = k[CH3CH2CH2Cl][c- OH]
The designation for this reaction is SN2, or a Substitution,
Nucleophilic, bimolecular (2) reaction.
The mechanism for the reaction of 1-chloropropane with water
is the concerted reaction shown in Section 12.2. With this mechanism,
the chloride ion leaves at the same time as the bond forms between
the sp3 carbon atom and the hydroxide ion. At that moment, the sp3
carbon atom is fully bonded to three substituents and partially bonded
to both the nucleophile and the leaving group.

H H
CH3CH2CH2Cl +

OH

HO

Cl

CH2CH3

CH3CH2CH2OH

Figure 6.3 is found on


page 000.

+ Cl

The reaction profile for the SN2 reaction looks similar to the
one in Figure 6.3. That profile shows only one transition state with no
intermediates between the reactants and products. The reaction
profile for the reaction of chloropropane with water, Figure 12.2, also
shows only one transition state indicating that the reaction depends
on the presence of both the substrate and the nucleophile.

www.ochem4free.com

5 July 2005

Organic Chemistry - Ch 12

594

Daley & Daley

H H
HO

Cl

CH2CH3

CH3CH2CH2Cl + OH

CH3CH2CH2OH + Cl

Reaction Progress
Figure 12.2. Reaction profile of the SN2 reaction.

Exercise 12.5
The reaction of ethyl bromide with the cyanide ion follows an SN2
pathway.
a) Write the rate equation for this reaction.
b) What happens to the rate of the reaction if the concentration of
cyanide ion is doubled? What happens to the rate of the reaction if
the concentrations of both cyanide and ethyl bromide are doubled?

12.4 Stereochemistry of Nucleophilic Substitutions

www.ochem4free.com

5 July 2005

Organic Chemistry - Ch 12

An inversion reaction
produces a product
with an opposite
stereochemistry from
the substrate.
Retention retains the
substrate
stereochemistry.

595

Daley & Daley

Chemists often use the product of an sp3 hybridized carbon


nucleophilic substitution reaction to identify which mechanism
operated in that particular reaction. When the reactive site is a
stereogenic center, a nucleophilic substitution reaction can yield
inversion, retention, or racemization of the configuration at the
asymmetric carbon. See Figure 12.3. In an inversion, the product has
the opposite stereochemistry of the substrate. In a retention, the
product has the same stereochemistry of the substrate. Most
nucleophilic substitution reactions with retention products are beyond
the scope of an introductory organic course, so this book does not cover
any examples of them. With racemization, equal amounts of both
retention and inversion occur.

Racemization produces
equal amounts of both.

X
Nu

X
C

Nu

Nu +

Inversion

X
Nu

X
C

C
Z

Retention

X
Nu

C
Z

X
Nu + Nu

C
Z

C
Z

+ L

Equal amounts
Racemization
Figure 12.3. The three possible stereochemical outcomes for a nucleophilic
substitution on an asymmetric, or chiral, substrate.

For more on the


VSEPR theory, see
Section 1.7, page 000.

Because the asymmetric carbon atom in an SN1 reaction loses


the leaving group before reacting with the nucleophile, the
intermediate carbocation that forms at the loss of the leaving group
has only three bonds to the carbon atom. According to the VSEPR
theory, these three bonds must lie in a plane with the carbon atom at
the center. With the carbon atom in that position, the intermediate is
symmetrical as shown in Figure 12.4. That symmetry allows the
nucleophile to react from either side of the carbon. Thus, an SN1
reaction produces a racemic product.

www.ochem4free.com

5 July 2005

Organic Chemistry - Ch 12

596

Daley & Daley

+
C
Y

Nu
Figure 12.4. The nucleophile can attack either side of the carbocation in an SN1
reaction.

For more on
hybridization, see
Section 1.7, page 000.

In an SN2 reaction, the leaving group departs from the


asymmetric carbon atom at the same time that the nucleophile
arrives. Because the leaving group blocks access to the front of the
carbon, the nucleophile must attack the back of the carbon. This
attack from the back of the carbon causes the molecule to invert.
Thus, SN2 reactions produce inversion products.
To understand the stereochemistry of an SN2 reaction,
reexamine the molecular orbital picture of an sp3 hybrid carbon
orbital. As you remember, an sp3 hybrid orbital has two lobes, one on
either side of the carbon atom, 180o apart. The two lobes are of
different sizes: a small lobe, which chemists often neglect to draw, and
a much larger lobe.
C
An sp3 hybrid orbital

A backside attack is an
attack from the
opposite side of the
carbon from the
leaving group.

The larger lobe is the lobe involved in bonding. It bonds the carbon to
the leaving group and, after the leaving group leaves, it bonds the
nucleophile to the carbon. However, the leaving group blocks the
nucleophile from having access to the larger lobe of this orbital. To
gain access to the larger lobe, the nucleophile attacks the back of the
carbon and literally turns the tetrahedron of the carbon atom inside
outlike an umbrella caught in the wind. Chemists call an attack on
the back of the carbon a backside attack. A backside attack,
illustrated in Figure 12.5, leads to the inversion of the configuration of
the carbon. Because the nucleophile reacts with the back of the carbon
bearing the leaving group, it is sensitive to the size of the other groups
on that carbon. Large groups tend to block access to the back of the
carbon atom.

www.ochem4free.com

5 July 2005

Organic Chemistry - Ch 12

597

Daley & Daley

X
Nu
C
Y

Figure 12.5. Attack by the nucleophile on the nonbonding lobe (backside) of the sp3
hybrid orbital.

A stereospecific
reaction has only one
stereochemical
outcome.
A stereoselective
reaction produces an
unequal mixture of
stereochemical
outcomes.

When a reaction produces a greater quantity of one


stereoisomer than it does of another, the reaction is either
stereospecific or stereoselective. An SN2 reaction is stereospecific
because its product is an inversion. A stereoselective reaction is
similar to a racemization except that it produces an unequal mixture
of stereochemical products.
Exercise 12.6
Draw the structure of the product expected from the SN2 reaction of
each of the following substrates with KOH.
a) (S)-2-Bromobutane

b)
Br
H

H
CH3

c)

d) (R)-3-Chlorohexane
H3C

H
C

Cl

Sample solution
b)
H
OH

H
CH3

12.5 The Substrate


www.ochem4free.com

5 July 2005

Organic Chemistry - Ch 12

598

Daley & Daley

A number of variables affect how an aliphatic nucleophilic


substitution reaction proceeds. These variables include the substrate,
the nucleophile, the leaving group, and the reaction medium. This and
the next several sections cover each of these variables in turn. After
presenting the general aspects about each variable, the section then
discusses how that variable particularly affects a nucleophilic
substitution at an sp3 hybridized carbon atom.
The first step in an SN1 reaction is the formation of a
carbocation from the substrate. The ease with which a carbocation
intermediate forms depends directly on the number of groups in the
substrate available to stabilize the positive charge of that carbocation.
When only alkyl groups are attached to the reactive carbon, the more
highly substituted the carbon, the more stable the carbocation. Thus,
the order for carbocation stability is 3o > 2o >> 1o > methyl. This order
is the reactivity order for an SN1 reaction, as well.
An alkyl group stabilizes the positive charge of a carbocation in
two ways: 1) through the inductive effect, and 2) through the field
effect. In the case of alkyl groups the inductive effect is simply the
donation of electron density through the V bonds from the alkyl groups
to the positively charged carbon. This effect distributes the positive
charge over several atoms. Thus, the more alkyl groups bonded to the
carbon, the greater the amount of donation that takes place.
Electron density is donated
from the carbons adjacent
to the carbocation.

C
C
C

Hyperconjugation is
the overlap of an empty
orbital on one atom
with a filled, bonding
orbital on an adjacent
atom.

The carbons in the alkyl groups bonded to the positively


charged sp3 hybridized carbon have atoms bonded to them, as well.
Through the field effect of the orbitals in those other atoms, the
carbons that they are bonded to donate electrons to the empty p
orbital of the positively charged carbon. This type of overlap, which is
called hyperconjugation, spreads the positive charge over additional
atoms beyond those affected by the inductive effect. The effect of
hyperconjugation is much weaker than the bonding overlap of orbitals,
but the combined inductive and field effects stabilize the positively
charged carbon of the carbocation. Figure 12.6 illustrates
hyperconjugation of a carbocation.

www.ochem4free.com

5 July 2005

Organic Chemistry - Ch 12

599

Daley & Daley

Overlap

Figure 12.6. In this example, hyperconjugation comes from the partial overlap of the
empty p orbital on the electron deficient carbon and the filled bonding molecular
orbitals of an adjacent carbonhydrogen bond.

A rearrangement
reaction is one where
the carbon skeleton
changes or the site of
the functional group
moves on the carbon
skeleton.

In a hydride shift, a
hydrogen on a carbon
adjacent to the
positively charged
carbon moves with its
bonding electrons from
its carbon to the
positively charged
carbon.
A methide shift is
similar to a hydride
shift. It occurs when
there are no hydrogens
to shift.

A carbocation often undergoes structural changes, called


rearrangements, to form a more stable carbocation. Rearrangements
occur either after the formation of the carbocation or as the leaving
group departs. If the rearrangement takes place as the leaving group
departs, the reaction is a concerted reaction. Because it is so much
higher in energy than secondary or tertiary carbocations, the
formation of a primary carbocation is very unlikely. Almost any time a
primary substrate is used, a rearrangement occurs in preference to
carbocation formation.
The driving force for the rearrangement of a carbocation is that
the product carbocation has a lower energy, or is more stable, than the
starting carbocation. For example, a primary or a secondary
carbocation rearranges to form a lower energy tertiary carbocation.
The rearrangement of the 3-methyl-2-butyl carbocation (secondary or
2o) to 2-methyl-2-butyl carbocation (tertiary or 3o) shows this.
H
CH3CHCCH3

H
CH3CHCCH3

CH3

CH3

The above reaction is also an example of a hydride shift.


A second type of rearrangement is a methide shift. When
there is no adjacent hydrogen, a methyl group with its pair of bonding
electrons moves from its carbon to the adjacent positive carbon. An
example is the rearrangement of the 2,2-dimethylpropyl (neopentyl)
carbocation to form the 2-methyl-2-butyl carbocation.
CH3
CH3CCH2
CH3

CH3CCH2CH3
CH3

www.ochem4free.com

5 July 2005

Organic Chemistry - Ch 12

A [1,2]-shift means
some group migrates
from one atom to an
adjacent atom.

600

Daley & Daley

In this example, the methide shift occurs while the leaving group is
leaving because the primary carbocation requires too much energy to
form directly. The energy difference between the primary and the
tertiary carbocation is approximately 12 kcal/mole. Both the hydride
and the methide shifts are part of a group of rearrangements called
[1,2]-shifts.
Exercise 12.7
Draw the structure of the product expected from an SN1 reaction of
each of the following substrates with H2O. Justify your product with a
mechanism.
a) 2-Bromo-3-methylbutane

b)
CH2Br

c)

d)
CH2Br

Br

Sample solution
c)
H

Br

OH

H2O
H

An SN2 reaction is very sensitive to crowding at the backside of


the substrate's sp3 hybridized carbon atom. Thus, the more steric
crowding present in a molecule, the slower is its rate of reaction. The
rate of reaction for methyl groups and most primary alkyl groups is
relatively rapid. For secondary groups the rate of reaction is generally
much slower. And most tertiary groups fail to have a reasonable

www.ochem4free.com

5 July 2005

Organic Chemistry - Ch 12

601

Daley & Daley

reaction rate at all. The reactivity order for the SN2 reaction is methyl
> 1o > 2o > 3o, which is the opposite of the reactivity order for the SN1
reaction.
The reactivity order for the SN2 reaction is due to the steric
hindrance of the larger groups. As the nucleophile moves to the back
of the carbon bearing the leaving group, it must pass close to the other
groups attached to that carbon. See Figure 12.7. (Please make models
to help yourself clearly understand these ideas.) With small groups,
such as hydrogens, the nucleophile reaches the back of the carbon
with little interference. However, with larger groups, methyl groups in
this example, the nucleophile must pass quite close to them to reach
the carbon. With three methyl groups, the area between them is often
smaller than the incoming nucleophile making it impossible for the
nucleophile to reach the backside of the carbon atom.
H

O-

Cl

Cl

O-

CH3

CH3
CH3
O-

Cl

CH3

Figure 12.7. Comparison of the steric hindrance of an SN2 reaction of hydroxide with
a methyl, primary, and tertiary halides.

Table 12.1 shows some examples of the effects of structure on


the rate of reaction via SN1 and SN2 reaction mechanisms. These
reaction rates are not absolute, but are based on the arbitrary
assignment of 1 as the rate for a methyl group.

www.ochem4free.com

5 July 2005

Organic Chemistry - Ch 12

602

Alkyl group

Daley & Daley

Relative Rate for


SN1 Reaction
1
1
12
1.2 x 106

CH3
CH3CH2
(CH3)2CH
(CH3)3C

Relative Rate for


SN2 Reaction
1
.03
.013
3 x 106

Table 12.1. Relative rates of reaction for SN1 and SN2 on different substrates with a
chloride leaving group.

Solved Exercise 12.1


Each of the following alkyl halide substrates is chiral and is present as a
single enantiomer. Show the stereochemistry of the product of the
nucleophilic substitution reaction on these alkyl halides.
a)

CN
KCN
H2O, CH3OH

Solution
The product of this reaction is a nitrile. Because iodide ion is a good leaving
group and the substrate is reasonably open sterically, the reaction follows an
SN2 mechanism. The product forms from an inversion reaction of the
substrate.

KCN

NC

H3C

OCH3

H2O, CH3OH
b)

CH3OH
H3C

Br

Solution
The substrate in this reaction is tertiary and follows an SN1 mechanism. An
SN1 reaction has a much higher rate than an SN2 reaction. The product
formed is a racemic mixture.

www.ochem4free.com

5 July 2005

Organic Chemistry - Ch 12

603

Daley & Daley

CH3OH
H3C

Br

H3C

OCH3

H3CO

CH3

c)

H
Cl

CH3CH2OH

OCH2CH3

Solution
This substrate easily forms a carbocation because the carbocation is
resonance-stabilized.

H
Cl

Thus, the product formed is a racemic mixture.


H

H
Cl

CH3CH2OH

OCH2CH3

OCH2CH3

+
H

OCH2CH3

OCH2CH3

Exercise 12.8
Rank the following compounds in order of increasing reactivity
towards SN1 substitution. Then rank them in order of increasing
reactivity towards SN2 substitution.

www.ochem4free.com

5 July 2005

Organic Chemistry - Ch 12

604

Daley & Daley

Br
(CH3)2 CHCH2Br

CH3CH2Br

(CH3CH2)3CBr

12.6 Nucleophiles and Leaving Groups

To abstract an atom
means to remove it.

In an SN1 reaction, the addition of the nucleophile to the sp3


hybridized carbon takes place after the leaving group departs.
Therefore, the nucleophile has no effect on the rate of the reaction
because the rate of addition of the nucleophile is much faster than the
rate of ionization. In an SN2 reaction, on the other hand, the addition
of the nucleophile to the sp3 hybridized carbon takes place as the
leaving group departs. So the nucleophile plays an important part in
the rate of reaction. Understanding nucleophiles will help you
understand how these reactions work.
Both nucleophiles and leaving groups are bases. In an aliphatic
nucleophilic substitution reaction the nucleophile is generally a
stronger base than the leaving group. Nucleophiles are good electron
donors and, in actuality, are Lewis bases. Chemists frequently
differentiate between nucleophiles and basesalthough those
differences are often fuzzy. Use the following as a working definition
to determine the differences between nucleophiles and bases: a base is
a compound that abstracts a proton, and a nucleophile is a compound
that reacts with any other electrophilic atom. A stronger base
abstracts the proton more rapidly, or more completely, than a weaker
base. A stronger nucleophile reacts with an electrophile more rapidly,
or more completely, than a weaker nucleophile. Under the appropriate
conditions, a Lewis base acts as either a base or a nucleophile.
When comparing similar species across the periodic table,
basicity, and nucleophilicity correlate. NH3 is both a stronger base
and a stronger nucleophile than H2O. And H2O is a stronger base and
a stronger nucleophile than HF. However, this generalization does not
apply when comparing atoms down the periodic table. As atomic
number increases within a family of elements, basicity decreases and
nucleophilicity increases. For example, H2S is a better nucleophile but
a weaker base than H2O. Figure 12.8 summarizes these trends.

www.ochem4free.com

5 July 2005

Organic Chemistry - Ch 12

605

Daley & Daley

Basicity

Nucleophilicity

Figure 12.8. Trends of nucleophilicity and basicity for similar species in polar
solvents, like water, with a proton attached to a heteroatom.

Recall from Section


5.3, page 000, that a
hard base is small and
nonpolarizable
whereas a soft base is
large and polarizible.

The concept of hard and soft acids and bases also helps
differentiate bases from nucleophiles because nucleophilicity increases
as a base becomes softer. For example, the ability of the halide ions to
act as leaving groups increases from fluoride (the hardest base in the
halogen family) to iodide (the softest base).
The sulfite ion (SO32c- ) demonstrates the differences between
bases and nucleophiles. The sulfite ion has a central sulfur with three
oxygen atoms bonded to it. Each oxygen atom bears a single formal
negative charge, and the sulfur bears a single formal positive charge:

In the presence of an acid, the sulfite ion acts as a base; in the


presence of an electrophile, it acts as a nucleophile. The oxygens of the
sulfite ion are more basic than the sulfur, so one of the oxygens will
react with a proton in acid to form the hydrogen sulfite (HSO3c- ), or
bisulfite, ion.

On the other hand, the sulfur is more nucleophilic than the oxygens;
thus, SO32c- reacts with methyl iodide to produce methyl sulfonate.
CH3

Methyl sulfonate

www.ochem4free.com

5 July 2005

Organic Chemistry - Ch 12

606

Daley & Daley

Even though the oxygens are negatively charged and are stronger
bases than the sulfur, the sulfur is a better nucleophile when reacting
with methyl iodide.
The size and shape of a compound also help determine whether
that compound is a nucleophile or a base. When the compound acts as
a base to pick up a small proton, its size is less important. However,
when the compound must act as a nucleophile and bond with a
tetrahedral carbon in an SN2 reaction, the size of the compound
determines whether or not it faces steric constraints.
The ethoxide ion and the tert-butoxide ion help illustrate how
the size and shape of a compound affect the basicity and
nucleophilicity of a compound. Both the ethoxide ion and the tertbutoxide are conjugate bases of alcohols and, of the two, the tertbutoxide ion is a slightly stronger base. However, the two compounds
are very different in size and shape. The ethoxide ion is much smaller,
and its shape leaves the oxygen much more available for bonding than
does the shape of the tert-butoxide ion. Therefore, the ethoxide ion
readily reacts with a carbon bearing a leaving group, but the tertbutoxide ion is too large for the backside attack, so it cannot function
as an effective nucleophile.
CH3
CH3

CH3CH2O

CO

CH3
Ethoxide ion

tert-Butoxide ion

Table 12.2 lists some relative rates for the SN2 reaction of
various nucleophiles with methyl iodide in DMF. Note that methyl
iodide does not have any steric crowding, thus, even large nucleophiles
readily react with it.
Nucleophile
CH3OH
FcCH3COOcClcNH3
(CH3)2S
N3cBrc-

Relative
Rate
1
5.0 x 102
2.0 x 104
2.4 x 104
3.1 x 105
3.6 x 105
5.9 x 105
6.1 x 105

Nucleophile
CH3OcCH3 CH2Occ- CN
(CH3) 3COc(CH3)2NH
CH3ScIc(CH3CH2)3P

Relative
Rate
2.0 x 106
4.3 x 106
5.0 x 106
7.0 x 106
1.1 x 107
1.6 x 107
2.1 x 107
3.0 x 109

Table 12.2. Relative rates for SN2 reaction in DMF with methyl iodide as the
substrate.

www.ochem4free.com

5 July 2005

Organic Chemistry - Ch 12

607

Daley & Daley

Exercise 12.9
In each of the following pairs of reagents, which is a better nucleophile
in an SN2 reaction with CH3Cl in a methanol solution?
a) CH3OH or CH3SH
c) Ic- or Brce) (CH3)3N or (CH3)3B

b) NH3 or NH4
d) CH3Oc- or CH3OH
f) (CH3)3P or (CH3)3N

Sample solution
c) Ic- is a better nucleophile than Brc- because Ic- is a softer
base than Brc- .
Recall from Chapter 8 that the best leaving groups are weaker
bases than the incoming nucleophile. To facilitate some reactions,
chemists use methods that convert poor leaving groups into good
leaving groups. For example, the leaving group of an alcohol is a
hydroxide ion. Because the hydroxide ion is a stronger base than many
nucleophiles, it is a relatively poor leaving group, which prevents
alcohols from readily undergoing nucleophilic substitutions. However,
when chemists add some acid to the alcohol, the hydroxyl group
protonates and becomes a better leaving group.

OH

A
R

OH

Chemists also use protonation to make ethers and some amines better
leaving groups. In each of these cases, the leaving group becomes a
neutral molecule, which is a much weaker base than the original
group. Water, alcohols, and some amines make very good leaving
groups.
A second method that chemists use to improve a group's ability
to leave is to convert it to some other functional group that is a better
leaving group than the original group. For example, an alcohol can
react with p-toluenesulfonyl chloride (usually called tosyl chloride) to
form a p-toluenesulfonate. p-Toluenesulfonate is a very good leaving
group because it is a weaker base than c- OH.

www.ochem4free.com

5 July 2005

Organic Chemistry - Ch 12

608

Daley & Daley

O
R

OH

CH3

SCl

CH3

HCl

Tosyl chloride

An alkyl tosylate

Usually, chemists abbreviate tosyl chloride as TsCl. They also write


the above reaction in the following way.
R

OH

TsCl

OTs

Solved Exercise 12.2


1-Propyl methanesulfonate is dissolved in acetone containing equivalent
amounts of both sodium chloride and sodium bromide. One product
predominates in the reaction. What is the product? Explain.

O
CH3CH2CH2

CH3

O
1-Propyl methanesulfonate
Solution
1-Bromopropane is the predominant product because bromide ion is a better
nucleophile than chloride ion (see Table 12.2) in an SN2 reaction on a
primary substrate.

12.7 Common Nucleophiles


Organic chemists use a variety of common nucleophiles. Table
12.3 lists some of these nucleophiles. This chapter discusses many of
these nucleophiles. You will see many of the others in nearly every
chapter in the rest of the text.
Nucleophile
ClcBrcIcH2O
HOcR'OH

Product
RCl
RBr
RI
ROH
ROH
ROR'

Nucleophile
R'ScNH3
R'NH2
R'2NH
R'3N
N3c(Azide ion)

www.ochem4free.com

Product
RSR'
RNH2
RNHR'
RNR'2
R NR'3
RN3
(Alkyl azide)

5 July 2005

Organic Chemistry - Ch 12

609

Daley & Daley

Nucleophile
R'OcR'COOH
R'COOcH2S

Product
ROR'
ROOCR'
ROOCR'
RSH

HSc-

RSH

C N

RSR'

O
C C

R'SH

Nucleophile
R'3P
HcR'3Cc-

R'C C

Product
RP R'3
RH
RCR'3

R C CR'
R C N

O
C C R

Table 12.3. The most common nucleophiles used in organic chemistry.

12.8 The Reaction Medium

A solvated ion has


solvent molecules
associated with it in
the solution.

Only rarely do chemists simply mix a substrate and reagent


together to run a reaction. Instead, they dissolve them in a solvent,
perhaps adding a catalyst, and then carefully adjust the temperature.
All these components make up the reaction medium, or environment,
in which a chemical transformation takes place. Of these various
components, the one that overwhelmingly influences the course of the
reaction is the solvent. The solvent has so much influence because it is
present in a much higher concentration than any other component. As
the reaction progresses, the solvent interacts with the substrate, the
nucleophile, the transition states, and the intermediates, thereby
influencing the rate of the reaction. Understanding the influence of
the solvent in a reaction helps you to know which solvent to choose
when you want to run a reaction. Understanding the influence of the
solvent also helps you to analyze the course of a reaction as you study
a reaction mechanism.
Solvent polarity, usually measured by the dielectric constant, is
a major factor in how a reaction proceeds. The dielectric constant is a
measure of the ability of the solvent to solvate, or stabilize, ions in
solution. Remember that the rate-determining step in an SN1 reaction
is the formation of the carbocation. More polar solvents accelerate the
rate of formation of a carbocation, thereby increasing the SN1 reaction
rates. The data in Table 12.4 show the ability of the more polar
solvents to accelerate the ionization of tert-butyl chloride.
Solvent
Water
Methanol

Structure
H2O
CH3OH

Dielectric
constant (H)
80
33

www.ochem4free.com

Relative rate
(at 25oC)
8 x 103
1 x 103

5 July 2005

Organic Chemistry - Ch 12

610

Daley & Daley

Solvent

Structure

Ethanol
Acetone
Ethyl ether
Pentane

CH3CH2OH
CH3COCH3
(CH3CH2)2O
CH3CH2CH2CH2CH3

Dielectric
constant (H)
24
21
4
2

Relative rate
(at 25oC)
2 x 102
1
1 x 103
< 104

Table 12.4. Dielectric constants for some representative solvents and their effect on
the rate of ionization for tert-butyl chloride.

With a solvent cage, the


positive ends of the
solvent molecules
associate around a
negative ion, or the
negative end of the
solvent molecules
associate around a
positive ion.

A polar solvent strongly solvates the ions formed in the SN1


reaction by building a solvent cage around each ion. In a solvent
cage, each molecule of solvent donates a little negative charge from its
nonbonded pair of electrons to the cation of the substrate. The higher
the dielectric constant of a solvent, the stronger its ability to solvate
ions in a reaction. Solvation lowers the energy of the ions and of the
transition state leading to them. For example, when the oxygen of
water donates its electron density to a cation, that cation becomes
solvated.
H
O

H
O

H
O
H

Solvated cation in water

Water can also solvate an anion. The anion donates electron density to
the partially positive hydrogens of the HO bond.

www.ochem4free.com

5 July 2005

Organic Chemistry - Ch 12

611

Daley & Daley

H
O
H
H

O
H

H
O

H
O
H
Solvated anion in water

Solvent polarity also affects SN2 reaction mechanisms.


However, because the mechanism does not involve ionization,
solvation of any intermediates is unimportant. The important factor is
the solvation of starting materials and products. Solvation of SN2
reactions fits into four categories. The difference between each
category depends on the charges of the starting materials.
The first category of SN2 reactions is the Type I SN2 reaction.
In a Type I SN2 reaction, the substrate and nucleophile are neutral,
and the initial product has a positive charge. The solvent builds a
solvent cage around the product; thus, lowering its energy level. With
a Type I SN2 reaction, a high polarity solvent actually causes a large
increase in the rate of the reaction.
Nu

Nu

SN2 Reaction - Type I

In the second category of SN2 reactions, the Type II SN2


reaction, the substrate is neutral and the nucleophile has a negative
charge. With a Type II SN2 reaction, the high solvent polarity
increases the solvation of the nucleophile. The solvent cage formed
around the nucleophile is so stable that it must disperse before the
nucleophile can react with the substrate. Thus, with the Type II SN2
reaction, increasing solvent polarity causes a small decrease in the
rate of reaction.
Nu

Nu

SN2 Reaction - Type II

In the third category of SN2 reactions, the Type III SN2


reaction, the leaving group has a positive charge and the nucleophile

www.ochem4free.com

5 July 2005

Organic Chemistry - Ch 12

612

Daley & Daley

has a negative charge. High solvent polarity increases the solvation of


the substrate and the nucleophile. The solvent builds a solvent cage
around both the substrate and the nucleophile, and both solvent cages
must disperse before the reaction can take place. With the Type III
SN2 reaction, increasing solvent polarity causes a large decrease in
the rate of reaction.
Nu

Nu

SN2 Reaction - Type III

The fourth category of SN2 reactions is the Type IV SN2


reaction. In a Type IV SN2 reaction, the leaving group has a positive
charge and the nucleophile is neutral. A high polarity solvent builds a
solvent cage around the substrate. Before the reaction can proceed
beyond this point, the nucleophile must penetrate this solvent cage.
With a Type IV SN2 reaction, increasing solvent polarity causes a
small decrease in the rate of reaction.
Nu + R

Nu

+ L

SN2 Reaction - Type IV

A polar-protic solvent
is a polar solvent
containing a
heteroatom bearing an
acidic proton.

A polar-aprotic solvent
is a polar solvent with
no acidic protons.

So far, all the solvents covered in this book are polar-protic


solvents. Polar-protic solvents have a heteroatom that bears a proton,
and they easily solvate both positive and negative ions. With polarprotic solvents, the anion becomes solvated by the hydrogen bonding
with the solvent. Polar-protic solvents work well as solvents for all
four types of SN2 reactions as well as SN1 reactions. Two common
examples of this type of solvent are water and methanol.
A second category of solvents commonly used in organic
chemistry is the polar-aprotic solvents. Even though polar-aprotic
solvents lack protons on a heteroatom, they can be quite polar. Thus,
they have the ability to solvate ions, even without the polar
hydrogens. Polar-aprotic solvents will solvate a nucleophile less than
does a polar-protic solvent, which gives the nucleophiles more freedom
to react. The rate for SN2 reactions in polar-aprotic solvents is highest
for types II and III. SN1 reactions generally do not proceed as well
with these solvents. Table 12.5 lists a few representative polar-aprotic
solvents.
Solvent
(Common abbreviation)

Structure

www.ochem4free.com

Dielectric
constant (H)

5 July 2005

Organic Chemistry - Ch 12

613

Solvent
(Common abbreviation)
Dimethyl sulfoxide
(DMSO)

Daley & Daley

Structure

Dielectric
constant (H)

O
CH3SCH3

45

Acetonitrile

CH3C

38

Dimethylformamide
(DMF)

O
HC

Hexamethylphosphoramide
(HMPA)

37

N(CH3)2

O
(CH3)2N

N(CH3)2

30

N(CH3)2
O

Acetone

21

CH3CCH3

Tetrahydrofuran
(THF)

7
O

Table 12.5. Some representative polar-aprotic solvents and their dielectric constants.

Polar-aprotic solvents promote a much faster rate of reaction in


SN2 reactions than do polar-protic solvents because polar-aprotic
solvents solvate only the positive ions. Solvating only the positive ion
leaves the anion unsolvated. These unsolvated, or naked, anions are
much more reactive than when solvated. For example, compare the
rate of reaction for the Type II reaction of azide ion (N3c- ) with methyl
iodide
using
methanol,
a
polar-protic
solvent,
versus
dimethylformamide (DMF), a polar-aprotic solvent.
CH3I

N3

CH3N3

Methanol and DMF have similar dielectric constants. The rate of


reaction in DMF solution, however, is 10,000 times larger than the
rate of reaction in the methanol solution.
Solved Exercise 12.3
When dissolved in hot ethanol, compound A is completely stable, but
compound B is rapidly converted to another compound. What is the product

www.ochem4free.com

5 July 2005

Organic Chemistry - Ch 12

614

Daley & Daley

of the reaction of ethanol with compound B? Explain the differences in


reactivity between compound A and compound B.

Br

Br
B

Solution
Compound B reacts with ethanol forming an ether.

Br

CH3CH2OH

OCH2CH3

This reaction occurs because compound B is a tertiary alkyl halide and


ethanol is a polar-protic solvent that promotes ionization of the substrate. On
the other hand, compound A is a primary substrate that is sterically blocked
for SN2 reaction. In addition, ethanol is not a strong enough nucleophile to
react with compound A.

Exercise 12.10
In water or methanol the order of halide nucleophile reactivity is Ic- >
Brc- > Clc- . However, with a solvent such as DMSO or DMF, the order
of reactivity reverses to Clc- > Brc- > Ic- . Explain this difference.
A crown ether is a
cyclic polyether having
an oxygen every third
atom around the ring.

The presence of a crown ether in a reaction essentially


cancels the effects of the solvent by complexing the positive counter
ion. Crown ethers, discovered in the 1960s, are cyclic polyethers. They
get their name from their three-dimensional crown shapes. Chemists
use crown ethers as complexing agents because of their high affinity
for various metal cations. As a complexing agent, a crown ether forms
a structure with the metal cation similar to a solvent cage. For
example, 18-crown-6 (18 is the ring size and 6 is the number of oxygen
atoms in the ring) binds strongly to a potassium ion.

www.ochem4free.com

5 July 2005

Organic Chemistry - Ch 12

615

Daley & Daley

O
O

O
O

Chelation is the
formation of a complex
between a metal ion
(the guest) and an
electron-donating
molecule (the host).

O
K
O

18-Crown-6

Potassium 18-crown-6 complex

Another word for the complex formed by a crown ether and a


metal ion is a chelation. A chelate (Greek chele, claw) is a binding
between a metal ion and an electron-donating molecule. 18-Crown-6 is
quite specific for the chelation of potassium whereas 15-crown-5
chelates sodium ions, and 12-crown-4 chelates lithium ions. In these
complexes, chemists call the crown ether the host and the metal ion
the guest.
Chelation makes ionic substances soluble in nonpolar organic
solvents where they are not usually soluble. In fact, after chelation,
most ionic substrates are much more soluble in nonpolar organic
solvents than they are in the solvents that they are soluble in without
the chelation. Because the crown ether does not complex with the
anion during a chelation, it leaves the anion readily available for
reaction. These anions, sometimes called naked anions because they
are not solvated, are much more reactive than they are when solvated.
Even poor nucleophiles become quite reactive. For example, the
reaction of benzyl bromide with potassium acetate yields less than
10% of the product, benzyl acetate. After adding 18-crown-6, however,
the yield jumps to 93%. The solvent used for the reaction is
acetonitrile (CH3CN), a polar-aprotic solvent that does not readily
dissolve ionic compounds.
O

CH2Br + CH3CO

Benzyl bromide

Potassium acetate

CH3COCH2
Benzyl acetate
(93%)

12.9 SN1 Versus SN2


So far, Chapter 12 has presented a number of factors that help
decide whether an SN1 or an SN2 substitution reaction takes place.
This section summarizes these factors in a series of tables. Table 12.6

www.ochem4free.com

5 July 2005

Organic Chemistry - Ch 12

616

Daley & Daley

compares the effects of various substrates on the outcome of the


reaction. Table 12.7 shows how the nucleophile/base, the leaving
group, and the solvent determine which mechanism a substrate will
follow.

CH3

CH2 L

CH

R
L

1o

Methyl
SN2 only.

2o
Mainly SN2 with weak
bases. Also SN1 in
acid, especially if the
substrate can
rearrange.

3o
No SN2. In a
solvolysis reaction
or under acidic
conditions, SN1.

Table 12.6. Summary of reaction pathways of substitution reactions by substrate


type.

Factor
Nucleophile
(base).
Leaving group.
Solvent.

SN1
Poor nucleophile.

SN2
Nucleophile is soft
base.
Better if a weaker base than the nucleophile.
Polar-protic solvent. Polar-aprotic solvent.

Table 12.7. Summary of effects of nucleophile, leaving group, and solvent on the
mechanism of a particular substrate.

12.10 Halide Nucleophiles

An oxonium ion is an
ether or alcohol with
the oxygen protonated.

In the previous discussions of aliphatic nucleophilic


substitution reactions, many of the examples used alkyl halides as the
substrate with the halide ion acting as the leaving group. Under the
appropriate circumstances, halides play a double role in aliphatic
nucleophilic substitutions: they act as the leaving group or they act as
the nucleophile. Chemists easily synthesize alkyl halides from
alcohols. They use these alkyl halides as reactive intermediates in
organic syntheses.
Chemists usually synthesize alkyl bromides via the reaction of
an alcohol with hydrogen bromide. Because the c- OH ion is a strong
base it is a very poor leaving group. However, the use of hydrogen
bromide allows protonation of the alcohol to form the oxonium ion.
The oxonium ion is an excellent leaving group because it is a relatively
weak base.

www.ochem4free.com

5 July 2005

Organic Chemistry - Ch 12

617

Daley & Daley

A
R

OH

OH

H
Oxonium ion

An SN2 reaction occurs with a primary or secondary substrate because


the Brc- nucleophile easily displaces the OH2 group. An SN1
reaction occurs with a tertiary substrate. First, the OH2 group
leaves to form the carbocation. Then the carbocation reacts with the
Brc- nucleophile in an SN1 reaction.
RCH2

RCH2

C
R

Br

SN2 synthesis of an alkyl bromide

R
R

Br

OH

OH

Br

Br

SN1 synthesis of an alkyl bromide

The Lucas Reagent is a


solution of ZnCl2
dissolved in
concentrated HCl.

Chemists prepare alkyl chlorides in a manner similar to the


way that they prepare alkyl bromides. However, hydrogen chloride is
a weaker acid than hydrogen bromide so formation of the oxonium ion
is less favorable with hydrogen chloride than it is with hydrogen
bromide. The chloride ion is also a weaker nucleophile than the
bromide ion, so the formation of the alkyl chloride occurs with a poorer
yield than does the alkyl bromide. Hydrogen chloride reacts well with
tertiary alcohols but much less satisfactorily with primary and many
secondary alcohols. Chemists often use the Lucas Reagent for
preparing primary and secondary halides because it adds the strong
Lewis acid ZnCl2 to the hydrogen chloride solution and enhances the
reactivity of the alcohols.
R

OH

ZnCl2

OH
ZnCl2

www.ochem4free.com

5 July 2005

Organic Chemistry - Ch 12

618

Daley & Daley

The resulting zinc chloride-alcohol complex is much more reactive


towards aliphatic nucleophilic substitutions than hydrogen chloride
alone. With the Lucas Reagent, the complex reacts with the chloride
ion to produce the alkyl chloride.

OH

Cl

Cl

ZnCl2

Synthesizing iodoalkanes is much more difficult than


synthesizing either alkyl chlorides or alkyl bromides. Generally,
iodoalkanes are too expensive and too reactive for widespread use.
They also often promote undesirable side reactions. When chemists do
use iodoalkanes, they synthesize them by reaction of a chloroalkane
with sodium iodide in acetone.
R Cl

The term in situ means


in place. A synthesis
done in situ completes
a reaction in the
reaction flask where
the product is to be
used without isolation.

NaI
acetone

This reaction is driven to completion because sodium chloride is


essentially insoluble in acetone, whereas sodium iodide is soluble in
acetone. The reaction follows SN2 kinetics.
In addition to the above methods for preparing alkyl halides,
chemists use thionyl chloride and phosphorus chloride to make alkyl
chlorides from alcohols. Because phosphorus bromide and phosphorus
iodide is so unstable, chemists usually synthesize them in situ using
phosphorus and bromine and iodine. In the mechanism for this
method, an ester of an inorganic acid forms, then the ester group
converts the alcohol to a good leaving group. The leaving group leaves
following an attack by the halide nucleophile.
R

OH

OH

OH

OH

SOCl2

PCl3

P, Br2

P, I2

Cl

Cl

Br

Below are some examples of alkyl halide synthesis.

www.ochem4free.com

5 July 2005

Organic Chemistry - Ch 12

619

HCl

OH

ZnCl2

Cl

Daley & Daley

Cl
1-Chloropentane
(81%)

NaI
Acetone

3-Methyl-1-iodobutane
(97%)

CH2OH

CH2Cl
SOCl2
Benzene
Pyridine

CH3

CH3
3-Methylbenzyl chloride
(89%)

Synthesis of 1-Bromobutane
NaBr
OH

H2SO4,

Br
1-Bromobutane
(78%)

Place 1.8 mL (24 mmol) of 1-butanol in a flask. Add 2.5 g (24 mmol) of sodium bromide
and 2 mL of water. Cool the flask in an ice bath and slowly add 1.6 mL of concentrated
sulfuric acid. Reflux this mixture for 45 minutes. Cool the apparatus to room
temperature. Remove the sulfuric acid layer and wash the organic layer with 5 mL of
water followed by 2 mL of saturated sodium bicarbonate. After separating the layers,
dry the organic layer with anhydrous sodium sulfate. Distill the product. Yield of 1bromobutane is 2.5 g (78%), bp 101-104oC.
Discussion Questions
1. Which layer would you expect to contain the 1-bromobutane, the upper or lower
layer? If, instead of 1-bromobutane, you prepared 1-chlorobutane, would the
organic layer be the same one (top or bottom)?
2. Does this reaction follow the SN1 or SN2 mechanism?
3. How would you expect this procedure to change if the reaction were run with 2methyl-2-propanol instead of 1-butanol? Would this reaction follow the SN1 or SN2
mechanism?

Exercise 12.11

www.ochem4free.com

5 July 2005

Organic Chemistry - Ch 12

620

Daley & Daley

Predict the major products of each of the following reactions.


a)
CH3CH2CH2CH2CHCH3

HBr

OH

b)
OH
HCl
ZnCl2

c)
PCl3
HO

d)
Cl
CHCH3

NaI
acetone

e)
CH2OH

SOCl2

f)
OH

P, I2

Sample Solution
e)
CH2OH

SOCl2

CH2Cl

Ethers are usually quite unreactive. To react, they require an


acid to protonate the oxygen and a good nucleophile. Two commonly
www.ochem4free.com

5 July 2005

Organic Chemistry - Ch 12

621

Daley & Daley

used examples are hot, concentrated solutions of HBr or HI. For


sterically open ethers, the reaction mechanism is SN2. For more
hindered ethers, the reaction pathway is SN1. The first step in either
pathway is the protonation of the ether oxygen.
H

Br

CH3OCH3

H
CH3O

CH3

The second step in an SN2 reaction is a nucleophilic substitution by


the halide ion.

H
CH3O

Br
CH3Br + CH3OH

CH3

The reaction of HBr with an ether forms an alkyl bromide and


an alcohol. The alcohol reacts with more HBr to form a second alkyl
bromide. However, this reaction is normally slow enough that the
alcohol can be isolated. In a reaction of an ether with HI, the iodide
ion is a better nucleophile than the bromide ion so the alcohol cannot
be isolated because it reacts so quickly with more HI. The following
reactions are examples of ether reacting with HBr and HI.
HBr

CH3CH2CH2OCH2CH2CH3

CH3CH2CH2Br
1-Bromopropane

OCH2CH3

CH3CH2CH2OH
1-Propanol

HI

Iodocyclohexane

CH3CH2I
Iodoethane

Exercise 12.12
In practice, an ether cleavage uses a large excess of HBr or HI.
However, the chemist observes no alcohol product and isolates only
the haloalkane. Write a mechanism to explain this observation.
An exception to the unreactivity of ethers is the oxirane ring.
This 3-membered ring is highly strained and very susceptible to
reaction with a variety of ring-opening reactions. For example, oxirane
ring ethers are among the few ethers that react with HCl directly to
cleave the ring.

www.ochem4free.com

5 July 2005

Organic Chemistry - Ch 12

622

Cl

O
CH3CH

Daley & Daley

CH2

HCl

CH3CHCH2OH
2-Chloropropanol
(98%)

The opening of the oxirane ring using HCl follows an SN2-like reaction
mechanism.
H

CH3CH

CH2

Cl

CH3CH

CH2

Cl

CH3CHCH2
OH

Cl

Unlike a typical SN2 reaction, the incoming nucleophile reacts at the


secondary carbon because it has a larger partial positive charge than
the primary carbon.
H

O
CH3CH

CH2

Larger partial positive


charge on this carbon.

Please note that under basic conditions in a ring opening reaction of


an oxirane, the nucleophile reacts with the least substituted carbon
atom.

12.11 Oxygen Nucleophiles


Aliphatic nucleophilic substitution reactions that involve an
oxygen atom nucleophile are very important in organic chemistry.
Oxygen nucleophiles fall into three major categories: water and
alcohols, hydroxide and alkoxide ions, and carboxylate ions. The
members of each category have about the same base strength, but
each category has a different base strength; thus, the particular
category of nucleophiles chosen for a reaction depends on the base
strength needed to accomplish that reaction. Water and hydroxide

www.ochem4free.com

5 July 2005

Organic Chemistry - Ch 12

In a solvolysis reaction,
the substrate reacts
with the solvent.

Elimination of alkyl
halides is discussed in
Section 13.7, page 000.

623

Daley & Daley

ions form alcohols, alcohols and alkoxide ions form ethers, and the
carboxylate ions form esters.
When studying the reaction mechanism of aliphatic
nucleophilic substitution, chemists used the reaction of alkyl halides
with water or hydroxide ion to form alcohols. However, this method is
not useful for the synthesis of alcohols because, as you studied in
Section 12.10, the best substrate to use when synthesizing an alkyl
halide is an alcohol. A hydride reduction of a carbonyl group or the
reaction of an organometallic with a carbonyl are better ways to
synthesize alcohols.
The SN2 reaction of water with primary alkyl halides and most
secondary alkyl halides is very slow and follows the reverse pathway
of the reaction forming alkyl halides from primary and secondary
alcohols. The SN1 reaction of tertiary alkyl halides and some
secondary alkyl halides with water is a more rapid reaction. This
increase in rate occurs because water is a polar-protic solvent and
promotes the ionization of tertiary and many secondary substrates.
The SN1 reaction of tertiary, and some secondary, halides with water
is an example of a solvolysis reaction. The mechanism of a
solvolysis reaction follows the reverse pathway of the reaction of a
halide nucleophile with an alcohol.
The hydroxide ion is a better nucleophile than water, so adding
it to the reaction mixture increases the rate of reaction. Adding
hydroxide ion also increases the rate of side reactions. The most
significant of which is a competing elimination reaction. In an
elimination reaction, the nucleophile reacts with a hydrogen on a
carbon adjacent to the leaving group taking it away. However, because
the hydroxide ion is a strong base a double bond forms rather than
another group substituting itself for the leaving group. Elimination
occurs because the hydroxide ion is not only a better nucleophile than
water but is also a moderately strong base. This basicity causes the
competing elimination reaction.
An elimination reaction is often a competing reaction for an
SN2 substitution reaction and even more frequently for an SN1
substitution reaction. An elimination reaction is an especially
significant competing reaction for reactions involving hydroxide ion
with alkyl halides. With these reactions, the proportion of competing
elimination reaction rises with the increase of steric crowding on the
substrate. For example, compare the percentages of the product
mixture in the reaction of the following alkyl bromide isomers with
hydroxide ion.

www.ochem4free.com

5 July 2005

Organic Chemistry - Ch 12

624

OH

CH3CH2CH2CH2CH2Br

CH3
CH3CH2CBr
CH3

OH
H2O

H2O

Daley & Daley

CH3CH2CH2CH2CH2OH

CH3CH2CH2CH CH2

1-Pentanol
90%

1-Pentene
10%

CH3
CH3CH2COH

CH3
+ CH3CH

CH3

CH3
2-Methyl-2-butanol

+ CH3CH2C

2-Methyl-2-butene

1%

CH2

CH3
1-Methyl-1-butene
5%

94%

Exercise 12.13
Write a mechanism for the solvolysis reaction of tert-butyl chloride
with water.
Alcohols are somewhat better nucleophiles than water. A
solvolysis reaction of alcohols with tertiary alkyl halides and some
secondary alkyl halides is a useful method for the synthesis of ethers.
CH3
CH3

Cl

CH3CH2OH

CH3

OCH2CH3

CH3
Ethyl tert-butyl ether
(89%)

CH3

In the Williamson ether


synthesis, an alkyl
halide reacts, in an SN2
reaction, with an
alkoxide ion forming
an ether.

CH3

The solvolysis of primary alkyl halides and most secondary


alkyl halides with alcohols does not generally work well. However, the
use of alkoxide ion, a stronger nucleophile, on these substrates does
make the reaction go. This reaction is an irreversible preparation for
ethers and is called the Williamson ether synthesis. The
Williamson ether synthesis works because an alcohol reacts with
sodium metal or some stronger base to form the alkoxide ion.
2 ROH

2 Na

2 RO Na

H2

The alkoxide ion then reacts with an alkyl halide to form an ether.
This reaction follows an SN2 reaction mechanism, as follows.

www.ochem4free.com

5 July 2005

Organic Chemistry - Ch 12

625

Daley & Daley

CH3CH2
O

CH3CH2 Br

CH3CH2

OCH
2CH3

Here is an example of the Williamson ether synthesis:

CH3CH2Br

CH3CH2CH2O Na

CH3CH2OCH2CH2CH3
Ethyl propyl ether
(70%)

This method produces cyclic ethers as well as acyclic ethers. To make


cyclic ethers, the hydroxide ion reacts with the alcohol to form an
alkoxide anion. The alkoxide anion then reacts with the alkyl halide
portion of the molecule forming the cyclic ether. Note that the alkoxide
ion is about the same base strength as the hydroxide ion, thus
establishing an equilibrium.
OH

NaOH

Cl

Cl

O
1,2-Epoxycyclohexane
(Cyclohexene oxide)
(77%)

OH

NaOH
O
Oxacyclohexane
(Tetrahydropyran)
(89%)

Exercise 12.14
Write a mechanism for the synthesis of oxacyclohexane shown in the
previous example.
Similar to the synthesis of alcohols, the Williamson ether
synthesis is also susceptible to a competing elimination reaction. The
reaction of a primary alcohol with a primary alkoxide favors a
substitution reaction. However, as steric hindrance increases, the
amount of competition from elimination reactions also increases. For

www.ochem4free.com

5 July 2005

Organic Chemistry - Ch 12

The acidity of phenol is


discussed in section
5.4, page 000.

626

Daley & Daley

example, if one of the reactants has a tertiary structure, the amount of


elimination reaction is often 30-40% of the amount of substitution
reaction. When both reagents have a tertiary structure, virtually no
substitution reaction occurs.
Phenols (pKa ~ 10) are much more acidic than are alcohols
(pKa ~ 17). Thus, the preparation of phenyl alkyl ethers requires a
much weaker base than does the preparation of alkyl ethers. Sodium
carbonate readily forms the phenoxide ion but is much too weak to
form an alkoxide ion. Thus, the synthesis of a phenyl alkyl ether
occurs with no side reaction. For example, chemists easily accomplish
the preparation of anisole by using sodium carbonate as a base.
OH

OCH3

CH3I
Na2CO3
Anisole
(92%)

The use of a polar-aprotic solvent increases the nucleophilicity


of the nucleophile. Often, this increase in nucleophilicity in turn
increases the yield of the substitution reaction product. For example,
the synthesis of dipropyl ether from sodium propoxide and 1chloropropane using 1-propanol as the solvent gives nearly a 70%
yield of product. Running the same reaction using dimethyl sulfoxide
(DMSO), a polar-aprotic solvent as the solvent increases the yield to
nearly 100%.
CH3CH2CH2O Na

+ CH3CH2CH2Cl

DMSO

(CH3CH2CH2)2O
Dipropyl ether
(99%)

Carboxylate ions are relatively poor nucleophiles. Their


alkylation with an alkyl halide, though not widely used, is a method
for the formation of esters. This reaction follows an SN2 substitution
reaction mechanism. When using carboxylate ions as the nucleophile,
chemists enhance their nucleophilicity by using a polar-aprotic
solvent. The example shown below uses dimethylformamide (DMF) as
the polar-aprotic solvent.
O

CH3CO K
CH3CH2CH2I

DMF

O
CH3COCH2CH2CH3
Propyl acetate
(96%)

www.ochem4free.com

5 July 2005

Organic Chemistry - Ch 12

627

Daley & Daley

Solvolysis is a very good method for the preparation of esters


using a tertiary alkyl group as the substrate. For this reaction,
chemists dissolve the alkyl halide in the carboxylic acid, then heat the
reaction mixture. In the example below, a good yield of the tertiary
alkyl ester forms.
CH3
CH3

CCl

O
CH3COH

O CH3
CH3COCCH3

CH3

CH3
tert-Butyl acetate
(81%)

Exercise 12.15
Predict the major products of each of the following reactions.
a)
CH2CH2Br

CH3O

b)
Cl
CH3CHCHCH3

NaOH

OH

c)
CH3

CH3OH

Br

d)
OH

CH3CH2I
Na2CO3

CH2OH

e)

www.ochem4free.com

5 July 2005

Organic Chemistry - Ch 12

628

Daley & Daley

1) Na
2) CH3I

OH

f)
Br

CH3CH2OH

CH2Br

Sample Solution
b)
Cl
CH3CHCHCH3

NaOH

CH3C H

CHCH3

OH

12.12 Nitrogen Nucleophiles


Chemists often synthesize amines via an SN2 reaction of
ammonia, or some other amine, with an alkyl halide substrate. The
reaction doesn't work well with tertiary halides because they are too
sterically hindered. Poor yields are often achieved with secondary
halides, too, because of significant amounts of competing elimination
reaction. The reaction does work well, however, with primary halides.
Although the example here shows chlorine, bromine and iodine are
also excellent leaving groups.

RNH2

+ R'CH2

Cl

RNH2CH2R' Cl
Salt of a secondary amine

Primary amine

As the reaction proceeds, it initially forms a non-nucleophilic


salt. This salt is in equilibrium with the original amine.

RNH2 +

RNH2CH2R' Cl

RNH3 Cl

+ RNHCH2R'

Thus, the secondary amine becomes a competitive nucleophile with


the original amine.

www.ochem4free.com

5 July 2005

Organic Chemistry - Ch 12

629

RNHCH2R' +

Daley & Daley

RNCH2R'

R'CH2Cl

CH2R'

The product of this step is also a nucleophile. It alkylates forming a


tetraalkyl ammonium salt.
CH2R'

RNCH2R' +

R'CH2Cl

RNCH2R' Cl
CH2R'

CH2R'

A tetraalkyl ammonium salt

The above reaction is useful under two conditions: 1) when


there is a large excess of the original nucleophilic amine (or ammonia)
to reduce the amount of competitive alkylation, and 2) when there is
an excess of the alkyl halide to exhaustively alkylate the nitrogen
when forming a tetraalkyl ammonium salt.
Under the first condition of adding a large excess of nucleophile
to the reaction mixture, chemists typically use ammonia (NH3) as the
nucleophile. Ammonia is cheap and relatively easy to handle. To run
the reaction, they add a primary alkyl halide to at least a ten-fold
excess of ammonia. The excess amount of ammonia effectively
eliminates dialkylation and higher alkylation products. At the end of
the reaction, they simply distill the ammonia from the reaction
mixture, leaving only the pure product behind.

CH2Br

NH3
(Excess)

CH2NH2
Aminomethylcyclopentane
(67%)

The Gabriel synthesis


involves reaction of the
anion of phthalimide
with an alkyl halide in
an SN2 reaction.

When doing an exhaustive alkylation to form the tetraalkyl


ammonium salt, chemists go to the other extreme. They add an excess
of the alkyl halide to the amine or ammonia. Often, they also add a
mild base, such as Na2CO3, to neutralize the large amount of HX
formed during the reaction.
Chemists have devised a variety of methods to overcome the
limitations of producing amines, particularly primary amines. Recall
the three methods presented earlier: the reaction of nitriles or amides
with a hydride nucleophile and the reaction of a ketone or aldehyde
with an amine and NaCNBH3. A fourth method is the Gabriel
synthesis. In the Gabriel synthesis, the nitrogen nucleophile is an
anion that readily forms from phthalimide in a reaction with KOH in

www.ochem4free.com

5 July 2005

Organic Chemistry - Ch 12

630

Daley & Daley

ethanol or sometimes water. In either case, the reaction forms a


resonance-stabilized anion.
O

O
KOH
Ethanol

NH

N
O

This resonance-stabilized anion, which is a good nucleophile, reacts


with a primary alkyl halide (usually a bromide) in an SN2 substitution
reaction.
O

O
N

RCH2Br

CH2R

The resulting imide then reacts with hydrazine to form the primary
amine and phthalimidehydrazide. This reaction, a nucleophilic
substitution at the carbonyl group, involves an amide exchange. An
amide exchange is the reaction of an amide with an amine to form
another amide and amine.
O

O
N

CH2R

NH2NH2

RCH2NH2

NH

NH
O

Below is an example of the Gabriel synthesis.

www.ochem4free.com

5 July 2005

Organic Chemistry - Ch 12

631

Daley & Daley

O
1) KOH, H2O
NH
2) CH3CH2CHCH2CH2Br

NH2NH2

CH3CH2CHCH2CH2NH2
CH3

CH3

3-Methyl-1-pentanamine
(88%)

Synthesis of 2,5-Diaminoadipic Acid

Br

CH3O

OCH3
O

CH3O
OCH3
O

Br

NH2NH2
NH2

NH2

HO
OH
O

HCl
H2O

CH3O

NH2

OCH3
O

NH2

2,5-Diaminoadipic acid
(80%)

Dimethyl 1,4-diphthalimidoadipate
Heat a mixture of 1.45 g (5 mmol) of dimethyl 2,5-dibromoadipate, 1.85 g (10 mmol) of
potassium phthalimide, and 5.2 mL of dimethylformamide. At 50oC a slightly
exothermic reaction begins. Maintain the temperature for 40 min at 90oC. Cool the
reaction mixture and add 6 mL of chloroform. Pour the mixture into 25 mL of water.
Separate the layers and wash the aqueous layer twice with 5 mL of chloroform. Wash
the combined chloroform extracts with 5 mL of 0.1M sodium hydroxide and 5 mL of
water. Dry the solution with anhydrous sodium sulfate. Remove the chloroform on a
rotary evaporator until crystals just begin to appear. Add 6 mL of ether and collect the
crystals. Wash the crystals with two 5 mL portions of cold ether. The product is
sufficiently pure for the next step. If you wish, you may recrystallize twice: first from
ethyl acetate and then from benzene.

www.ochem4free.com

5 July 2005

Organic Chemistry - Ch 12

632

Daley & Daley

2,5-Diaminoadipic acid
To 1.75 g of crude product from the above step, add 20 mL of methanol and 0.45 mL of
85% aqueous hydrazine solution. Reflux for 1 hour. Cool the reaction mixture and add
7.5 mL of water. Remove the methanol on a rotary evaporator. Add 7.5 mL of
concentrated hydrochloric acid. Reflux for 1 hour. Cool to 0oC and filter the solution.
On a rotary evaporator concentrate the aqueous solution until a moist solid remains.
Dissolve the solid in 15 mL of water and neutralize (pH ~ 6) the solution with 2M
aqueous sodium hydroxide. Filter out any insoluble material. Cool the resulting
solution for 12 hours at 0oC, then collect the crystals. Yield 0.53 g (60%) of product,
mp decomposes.
Discussion Questions
1. After refluxing with hydrazine, you remove and discard a solid material. What is
the structure of that solid material?
2. Why must you adjust the pH of the solution of 2,5-diaminoadipic acid to 6 to
crystallize the product?

Another common nitrogen nucleophile is the azide (N3c- ) ion.


An azide ion, the conjugate base of the acid HN3, is a resonance hybrid
of the following structures:
2

The anion is relatively non-basic (the pKa of HN3 is 11), but it is a


good nucleophile and reacts readily in an SN2 reaction to form alkyl
azides.
Br

NaN3
CH3OH, H2O

N3
1-Butyl azide
(86%)

Alkyl azides are reduced with LiAlH4 to produce the corresponding


amines. This reaction provides a convenient synthesis for amines.
N3

NH2

1) LiAlH4
2) H3O

Cyclohexanamine
(94%)

Exercise 12.16

www.ochem4free.com

5 July 2005

Organic Chemistry - Ch 12

633

Daley & Daley

Predict the major products of each of the following reactions.


a)
CH3CHCH2CH2Br
CH3

NH3
Large excess

b)
NH2

CH3I
Excess

c)
NaN3
Br

d)
O
1) KOH

NH
O

CH2Cl

2)
3) NH2NH2

e)
CH3CH2Br
N
CH3

f)
O
NH
O

1) KOH
2) BrCH2CH2CH2Br
3) NH2NH2

Sample Solution
a)

www.ochem4free.com

5 July 2005

Organic Chemistry - Ch 12

634

NH3

CH3CHCH2CH2Br

Large excess

CH3

Daley & Daley

CH3CHCH2CH2NH2
CH3

12.13 Carbon Nucleophiles


The study of stabilized carbanions in Chapter 19 considers
carbon nucleophiles in greater detail than this section does. However,
several important reactions also fit in this chapter. These reactions
use the cyanide ion nucleophile, the acetylide ion, and the
organometallics.
Because the cyanide ion is similar to the iodide ion in
reactivity, the cyanide ion is a good nucleophile. Cyanide ion
substitutions proceed through the SN2 mechanism, so the cyanide ion
is useful with primary substrates, as well as many secondary
substrates.
Cl

KCN
H2O

CN
Hexanenitrile
(81%)

Br

CN

NaCN
H2O

Cyclohexanecarbonitrile
(67%)

A phase-transfer
catalyst is a compound
that carries some
chemical species into a
solvent in which it is
otherwise insoluble.

A cyanide ion substitution reaction often proceeds with


difficulty because both sodium and potassium cyanides are insoluble
in most organic solvents and the substrates are insoluble in water.
Using crown ethers, or some other member of this group of compounds
called phase-transfer catalysts, helps to overcome these problems.
For example, crown ethers carry a variety of negatively charged
nucleophiles into nonpolar solvents.
Other widely used phase-transfer catalysts are quaternary
ammonium salts (R4N Xc- ). Quaternary ammonium salts often show
varying solubilities in different solvents with changes in the anion.
For example, tetrabutyl ammonium chloride is quite soluble in water
but shows low solubility in many organic solvents. On the other hand,
tetrabutyl ammonium cyanide has a higher solubility in organic
solvents. In water that contains the cyanide ion, the tetrabutyl

www.ochem4free.com

5 July 2005

Organic Chemistry - Ch 12

635

Daley & Daley

ammonium chloride and the tetrabutyl ammonium cyanide are in


equilibrium.
Bu4N Cl

Bu4N

CN

CN

Cl

Once in the organic solvent, the cyanide ion reacts with the substrate,
releasing the halide ion. The tetrabutyl ammonium halide then
migrates to the aqueous phase ready to carry another cyanide ion to
the organic phase.
Bu4N

An acetylide ion is the


conjugate base formed
by reaction of a strong
base with a terminal
alkyne.

CN

Bu4N

RCH2Cl

Cl

RCH2CN

When they react with a strong base such as sodium amide


(NaNH2) or Grignard reagents, terminal alkynes ( RC CH) have
sufficient acidity to form carbanions, also called acetylide ions. The
ease of formation of the acetylide ion is due to the sp hybrid orbital of
the triple bond carbon. Because this bond is shorter than the sp2 or
sp3 carbon to hydrogen bonds, it is also weaker. Acetylide ions are
good nucleophiles and react readily in SN2 substitution reactions.
RC

CH

RC

CH

NaNH2

RC

Na

RC

MgBr

NH3 (33oC)

R'MgBr
ether

Chemists prepare sodium amide by adding sodium to liquid ammonia


in the presence of catalytic quantities of the iron(III) cation. This
process is colorful. The solution of sodium in liquid ammonia is an
intense deep blue color. As the sodium reacts with the mixture, it
changes to a suspension of a gray solid in a colorless liquid. At this
point, the chemists add the terminal alkyne, followed by the
alkylating agent. The reaction mixture makes no further appreciable
color change.
RC

Na

R'CH2Br
NH3 (33oC)

R'CH2C

CR

Grignard reagents and organolithium reagents are not usually


good sources of carbon nucleophiles in SN2 substitution reactions.
They are such strong bases that they tend to promote elimination
reactions instead. However, they do substitute on some particularly

www.ochem4free.com

5 July 2005

Organic Chemistry - Ch 12

636

Daley & Daley

reactive substrates. For example, epoxides are strained cyclic ethers


that react with both Grignard reagents and organolithium reagents.
This reaction is notable in that it results in the extension of the carbon
chain with a reactive functional group on the second carbon.
MgCl

1)

CH2CH2OH

2) H3O
2-Cyclohexylethanol
(69%)

Exercise 12.17
Propose a synthesis for the following compounds.
a) 4-Nonyne
c) 2-Cyclohexyl-1-pentanol
e) 2-Phenylethanol

b) 3-Methylhexanol
d) 1-Decyne

Sample Solution
a) There are two possible ways to synthesize this compound.
1)
CH3CH2CH2CH2C CH

1) NaNH2/NH3
2) CH3CH2CH2Br

CH3CH2CH2C

CCH2CH2CH2CH3

2)
CH3CH2CH2C

CH

1) NaNH2/NH3
2) CH3CH2CH2CH2Br

CH3CH2CH2C

CCH2CH2CH2CH3

12.14 Neighboring Group Participation

With neighboring
group participation, a
functional group in the
substrate, other than
the leaving group,
affects the outcome of
the reaction.

With the exception of the formation of cyclic ethers, the


substitution reactions covered to this point have involved separate
substrates and nucleophiles. With the formation of cyclic ethers,
nucleophilic substitution can also be an intramolecular process. In an
intramolecular reaction, called neighboring group participation,
the presence of functional groups in the substrate other than the
leaving group affects the rate and/or the stereochemistry of the
reaction. Thus, when the nucleophile and leaving groups are a part of
the same molecule, the result is a cyclic compound.

www.ochem4free.com

5 July 2005

Organic Chemistry - Ch 12

637

Cl

OH

Daley & Daley

NaOH
O
Tetrahydropyran
(89%)

NH2

HN

Cl

NaOH
CH3CH2OH

2,3-Diphenylaziridine

(94%)

Intramolecular processes have a more favorable entropy than


do analogous intermolecular processes because the two separate
molecules are not required to come together. Five- or six-membered
rings form with reaction rates that are typically about 106 faster than
comparable acyclic reactions. Even a three-membered ring forms
about 103 faster than a similar acyclic process.
Formation of a three-membered ring is due more to proximity
effects than entropy. For example, the formation of an epoxide from a
chlorohydrin requires only a small movement of the atoms for bond
formation. The oxygen anion is about 250 pm away from the carbon
bearing the chlorine. The length of a CO bond is about 150 pm.
250 pm
150 pm

CH2

C
H

Anchemeric assistance
results in a rate
enhancement in a
reaction involving
neighboring group
assistance.

Cl

CH2

C
H

Neighboring group participation in a reaction is not always as


obvious as in the above reactions. Often the only obvious effect is a
marked rate enhancement over similar reactions with no neighboring
group participation. Chemists call such rate enhancements
anchemeric assistance (Greek anchi + meros meaning neighboring
parts). For example, the hydrolysis reaction of E-chloroethyl ethyl
sulfide is 3,000 times faster than the hydrolysis of 1-chloropentane.

www.ochem4free.com

5 July 2005

Organic Chemistry - Ch 12

638

Daley & Daley

CH3CH2SCH2CH2Cl + H2O

CH3CH2SCH2CH2OH + HCl

-Chloroethyl ethyl sulfide

-Hydroxyethyl ethyl sulfide

CH3CH2CH2CH2CH2Cl + H2O

CH3CH2CH2CH2CH2OH +

HCl

1-Pentanol

1-Chloropentane

The first reaction is a first order reaction. Its rate depends only on the
concentration of the substrate. The concentration of the nucleophile
has no effect on the rate. This is surprising because the second
reaction follows second order kinetics.
The difference between the two reactions above must have
something to do with the sulfur atom, but the sulfur is too far from the
reaction site to electronically affect the rate of the reaction. However,
sulfur is a good nucleophile and the chloride ion is a good leaving
group, so the sulfur forms a cyclic sulfonium salt by displacing the
chloride ion.

CH3CH2

CH2CH2 Cl

Cl

CH3CH2

CH2
CH2

Sulfonium ion

The intermediate sulfonium ion is very reactive with an incoming


nucleophile because the positive sulfur is a good leaving group. Thus,
the nucleophile reacts with the sulfonium ion, which is not an
ordinary primary alkyl halide.

CH3CH2

CH2
CH2

H2O
(-H )

CH3CH2

CH2CH2OH

Solved Exercise 12.4


Often chemists label a compound in an experiment with an uncommon
isotope of one of its atoms to use that atom as a probe to understand the
mechanism of the reaction. The accelerated rate of the nucleophilic
substitution reaction of E-chloroethyl ethyl sulfide is an interesting example
of the use of labeling to probe the reaction. If the carbon bearing the chlorine
is labeled with 14C, what labeling pattern would be observed in the product if
a) there is no neighboring group participation or b) neighboring group
participation occurs?

www.ochem4free.com

5 July 2005

Organic Chemistry - Ch 12

639

Daley & Daley

14

CH3CH2SCH2 CH2Cl
Labeled -chloroethyl ethyl sulfide
Solution
a) If there is no neighboring group effect, then the product will be labeled on
the carbon bearing the OH group.
14

CH3CH2SCH2 CH2OH
b) If neighboring group participation is important in this reaction, then there
will be a 1:1 mixture with the labeled carbon either bearing the OH group
or attached to the sulfur.

Cl

CH3CH2

14

CH3CH2SCH2 CH2OH

Water can react equally


well with either carbon.

14

CH2
CH2

14

CH3CH2S CH2CH2OH

The actual result is a 50:50 mixture of the two label positions produced as in
part b).

Exercise 12.18
The rate of hydrolysis of E-chloroethyl ethyl ether is much slower than
the rate of hydrolysis of E-chloroethyl ethyl sulfide. Explain this
observation.

Special TopicSN1 Versus SN2


The SN1 and SN2 reaction mechanisms represent the two
extremes of nucleophilic substitution reactions at an sp3 saturated
carbon. With an SN1 reaction, the reaction rate depends only on the
substrate because the ionization of the bond between the electrophilic
carbon and the leaving group is a slower step than the reaction with
the nucleophile. With an SN2 reaction, the rate of reaction depends on
the concentrations of both the substrate and the nucleophile because
the bond breaking and bond formation occur simultaneously.

www.ochem4free.com

5 July 2005

Organic Chemistry - Ch 12

Borderline behavior
occurs when a
nucleophilic
substitution reaction
does not follow either
SN1 or SN2 kinetics.

A solvent-separated ion
pair has a few
molecules of solvent
between a pair of ions.

Daley & Daley

Most aliphatic nucleophilic substitution reactions fall


somewhere between these two extremes. Some unimolecular processes
occur with a partial amount of inversion, but some of the bimolecular
processes exhibit less than 100% inversion. These in-between
reactions exhibit what chemists call borderline behavior. The
reaction is not strictly an SN1 or an SN2 reaction but shows some
components of both mechanisms. Where a particular reaction falls in
the continuum between SN1 and SN2 mechanisms depends largely on
the substrate; although the solvent, nucleophile, and leaving group
also play a part.
Richard A. Sneen of Purdue University suggested a unified
theory that explains the pure SN1 and SN2 mechanisms as well as
the borderline behavior (Acc Chem. Res., 6, 46-53 (1973)). His theory,
simply put, says that every substrate first ionizes to form an
intermediate ion pair. That ion pair then converts to products.

RL

A caged ion pair is a


pair of ions surrounded
by solvent molecules.

640

R L

Nu

RNu

According to his theory, the rate-determining step of the two equilibria


above is the factor that determines whether a particular reaction
follows an SN1 or an SN2 mechanism. If the formation of the ion pair
determines the rate, then the reaction follows SN1 kinetics. If the
reaction of the ion pair determines the rate, then the reaction follows
SN2 kinetics. Reactions following borderline behavior involve both
steps in determining the reaction rate.
An important consideration for both SN1 and SN2 reaction
mechanisms is the actual steps in ionization. As ionization occurs, a
cage of solvent molecules surrounds the pair of ions. Chemists often
call this formation a caged ion pair. A caged ion pair is essentially
the same as a solvent cage, except a caged ion pair involves a pair of
ions, whereas a solvent cage involves only one ion. Shortly after
ionization occurs, only a few solvent molecules separate the two ions,
and the ion pair is called a solvent-separated ion pair. Finally, the
ions become completely separated and exist as free solvated ions.
Figure 12.9 depicts the steps of this process.

www.ochem4free.com

5 July 2005

Organic Chemistry - Ch 12

RL

H2O

641

R L

H
R

O
H

H
O

H2O

H
O

H
H

H
H

Daley & Daley

H
Solvent-separated ion pair

Caged ion pair


H2O
H

H
O

H
O

H
R

O
H

H
O

H
Free ion pair

Figure 12.9. Schematic of the ionization process.

With a high polarity solvent, the substrate tends to ionize


further than with a low polarity solvent. Ionization that occurs with a
high polarity solvent in combination with a substrate that forms a
more stable carbocation, or has a very good leaving group, tends to
show SN1 kinetics. A low polarity solvent discourages ionization. A
substrate that does not form a stable carbocation, or that has a poor
leaving group, shows SN2 kinetics.
The most important factor in the kinetics of an aliphatic
nucleophilic substitution is the substrate's structure and its ability to
ionize. Nearly as important is the ability of the solvent to promote
ionization. Of somewhat less importance is the ability of the leaving
group to leave. In relation to the leaving group, the more SN2
character that a reaction possesses, the greater is the importance of
the strength of the nucleophile. Only when the nucleophile is a
stronger base than the leaving group does the equilibrium favor the
product.
When a reaction mixture consists of a primary substrate in a
low polarity solvent that does not promote ionization and a good
nucleophile, then the reaction favors the SN2 reaction mechanism.
Methyl compounds react the fastest. Primary compounds react nearly
as fast and secondary compounds less so. Tertiary compounds do not
react via the SN2 mechanism.

www.ochem4free.com

5 July 2005

Organic Chemistry - Ch 12

642

Daley & Daley

Solvents that promote the formation of free ions from the


substrate, tertiary substrates, or other compounds that ionize readily,
the SN1 reaction mechanism is favored. The intermediate carbocation
in an SN1 reaction is a strong Lewis acid. Acidic reaction conditions
encourage the formation of the intermediate carbocation, but in a
basic reaction medium, carbocations do not form. If the reaction
medium is basic and the nucleophile is a strong enough base, the
reaction prefers an elimination mechanism.
Exercise 12.19
Predict the major products of each of the following reactions.
Determine whether the reaction is primarily SN1 or SN2.
a)
CH3

HCl
ZnCl2

COH
CH3

b)
CH3C
Br

CNa

NH3

c)
CH2OTs

NaN3
DMF

d)
Br

CH3CH2OH

CH3

e)
CH3
CH3C
CH3

NH2

CH3CH2I
(excess)

Sample Solution
c) This is an SN2 reaction.

www.ochem4free.com

5 July 2005

Organic Chemistry - Ch 12

CH2OTs

643

NaN3
DMF

Daley & Daley

CH2N3

Key Ideas from Chapter 12


An sp3 hybridized carbon undergoes nucleophilic substitution if
it has a bond to an atom more electronegative than itself.
Nucleophilic substitution at a saturated carbon atom follows
one of two mechanisms. In the first mechanism, the leaving
group departs before the nucleophile arrives. This is the SN1
mechanism. In the second mechanism, the leaving group
departs as the nucleophile arrives. This is the SN2 mechanism.
The rate for an SN1 mechanism depends only on the
concentration of the substrate. Thus, an SN1 reaction follows
first order kinetics and proceeds through a carbocation
intermediate.
The rate for an SN2 mechanism depends on the concentrations
of both the nucleophile and the substrate. Thus, an SN2
reaction follows second order kinetics and is a concerted
reaction.
Because the SN1 mechanism has a symmetrical carbocation
intermediate, it loses all stereochemical information in the
reaction. The reaction proceeds with racemization of
configuration.
In the SN2 reaction, the nucleophile approaches from behind
the leaving group resulting in an inversion of the configuration
of the carbon in the substrate. In the inversion of configuration
of the SN2 mechanism, the product has the opposite
configuration of the starting material.
The SN1 mechanism works best in reactions that produce the
most stable carbocations. Tertiary carbocations are more stable
than either secondary or primary carbocations because of
hyperconjugation.
When a primary or secondary carbocation forms, it rearranges,
if possible, to form a tertiary carbocation.

www.ochem4free.com

5 July 2005

Organic Chemistry - Ch 12

644

Daley & Daley

The SN2 mechanism works best in reactions where the


structure of the substrate gives the nucleophile access to the
backside of the electrophilic carbon atom. Thus, the highest
rate of reaction is with methyl, or primary, substrates.
Nucleophiles and leaving groups are both bases. Usually, the
leaving group is a weaker base than the nucleophile. Most good
nucleophiles are soft bases.
Reactions with both high solvent polarity and the ability of the
solvent to solvate both the carbocation and the nucleophile
promote SN1 reaction pathways.
Low solvent polarity that does not stabilize carbocation
formation promotes SN2 reaction pathways.
Crown ethers and quaternary ammonium salts are examples of
phase-transfer catalysts. A phase-transfer catalyst changes an
ionic species that is otherwise insoluble in a low polarity
solvent to one that is soluble. This change makes a nucleophile
much more reactive.
Halide nucleophiles react via either the SN1 or the SN2
mechanism depending on the substrate and reaction
conditions.
Water and alcohols are good nucleophiles for solvolysis
reactions. Their conjugate bases are good nucleophiles as well,
but they tend to promote elimination reactions as side
reactions to substitution reactions.
Ammonia and amines are excellent nucleophiles in SN2
reactions.
Cyanide and acetylide ions are also good nucleophiles in SN2
substitution reactions.
Most nucleophilic substitutions are neither pure SN1 nor SN2
reactions but fall somewhere in between. An ionization process
has been proposed to explain the SN1-SN2 continuum. The
closer together an ion pair stays, the more SN2 characteristics
that reaction possesses. The further they move apart, the more
SN1 character the reaction shows.

www.ochem4free.com

5 July 2005

Das könnte Ihnen auch gefallen